{ "cells": [ { "cell_type": "markdown", "metadata": {}, "source": [ "*This notebook was created by Sergey Tomin (sergey.tomin@desy.de). Source and license info is on [GitHub](https://github.com/ocelot-collab/ocelot). June 2018.*" ] }, { "cell_type": "markdown", "metadata": {}, "source": [ "### Appendix: FODO in thin lens approximation \n", "\n", "*This notebook was created as an answer for a question what the min/max average beta function can have the EuXFEL SASE1 undulator. The right way is asking the optics expert but it is too simple and does not make you feel the beam optics of this SASE line. Another problem is that FEL experts like to work with average betas. To meet all these requirements, I wrote this notebook, which also might help someone to understand how to deal with linear optics in OCELOT.*\n", "\n", "Let's consider the FODO cell of the SASE undulator \n", "\n", "\n", "\n", "Introducing $d = L_{cell}/2$ as the distance between defocusing and focusing quadrupoles.\n", "\n", "We neglect focusing effect in the vertical plan of the horizontal planar undulator.\n", "\n", "The transfer matrix of the quadrupole in thin lens approximation:\n", "$$\n", "\\begin{equation}\n", "M_f = \n", "\\begin{bmatrix}\n", " 1 & 0 \\\\\n", " 1/f & 1 \n", "\\end{bmatrix}\n", "\\end{equation}\n", "$$\n", "where $f = 1/kl$ is focal length, $k$ is quad strength and $l$ is the quad length. \n", "\n", "Drift:\n", "$$\n", "\\begin{equation}\n", "M_d = \n", "\\begin{bmatrix}\n", " 1 & d \\\\\n", " 0 & 1 \n", "\\end{bmatrix}\n", "\\end{equation}\n", "$$" ] }, { "cell_type": "code", "execution_count": 1, "metadata": {}, "outputs": [], "source": [ "%matplotlib inline\n", "import sympy as sp \n", "sp.init_printing()" ] }, { "cell_type": "markdown", "metadata": {}, "source": [ "\n", "Calculate the matrix for a half cell, starting in the middle of a defocusing quadrupole and ending in the meddile of a focusing lens. \n", "\n", "And denotes $f_h = 2 f$ focal length of the half quadrupole" ] }, { "cell_type": "code", "execution_count": 2, "metadata": {}, "outputs": [ { "data": { "image/png": "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", "text/latex": [ "$\\displaystyle \\left[\\begin{matrix}\\frac{d + fh}{fh} & d\\\\- \\frac{d}{fh^{2}} & \\frac{- d + fh}{fh}\\end{matrix}\\right]$" ], "text/plain": [ "⎡d + fh ⎤\n", "⎢────── d ⎥\n", "⎢ fh ⎥\n", "⎢ ⎥\n", "⎢ -d -d + fh⎥\n", "⎢ ─── ───────⎥\n", "⎢ 2 fh ⎥\n", "⎣ fh ⎦" ] }, "execution_count": 2, "metadata": {}, "output_type": "execute_result" } ], "source": [ "fh, d = sp.symbols(\"fh, d\")\n", "\n", "Mqf = sp.Matrix([[1, 0],[-1/fh, 1]])\n", "Md = sp.Matrix([[1, d],[0, 1]])\n", "Mqd = sp.Matrix([[1, 0],[1/fh, 1]])\n", "M1 = Mqf*Md*Mqd\n", "sp.simplify(M1)" ] }, { "cell_type": "markdown", "metadata": {}, "source": [ "For second half cell $f_h \\to -f_h$" ] }, { "cell_type": "code", "execution_count": 3, "metadata": {}, "outputs": [ { "data": { "image/png": "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", "text/latex": [ "$\\displaystyle \\left[\\begin{matrix}- \\frac{2 d^{2}}{fh^{2}} + 1 & \\frac{2 d \\left(d + fh\\right)}{fh}\\\\\\frac{2 d \\left(d - fh\\right)}{fh^{3}} & - \\frac{2 d^{2}}{fh^{2}} + 1\\end{matrix}\\right]$" ], "text/plain": [ "⎡ 2 ⎤\n", "⎢ 2⋅d 2⋅d⋅(d + fh)⎥\n", "⎢ - ──── + 1 ────────────⎥\n", "⎢ 2 fh ⎥\n", "⎢ fh ⎥\n", "⎢ ⎥\n", "⎢ 2 ⎥\n", "⎢2⋅d⋅(d - fh) 2⋅d ⎥\n", "⎢──────────── - ──── + 1 ⎥\n", "⎢ 3 2 ⎥\n", "⎣ fh fh ⎦" ] }, "execution_count": 3, "metadata": {}, "output_type": "execute_result" } ], "source": [ "M2 = sp.Matrix([\n", "[(d - fh)/-fh, d],\n", "[ -d/fh**2, (-d -fh)/-fh]])\n", "M_cell = sp.simplify(M1*M2)\n", "M_cell" ] }, { "cell_type": "markdown", "metadata": {}, "source": [ "#### Phase advance and stability of the FODO cell\n", "\n", "Phase advance is related to the transfer matrix by:\n", "$$\n", "\\cos \\phi_{cell} = \\frac{1}{2}Trace(M_{cell}) \n", "$$\n", "Stability requares:\n", "$$\n", "\\left|Trace(M_{cell})\\right| < 2\n", "$$" ] }, { "cell_type": "code", "execution_count": 4, "metadata": {}, "outputs": [ { "data": { "image/png": "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", "text/latex": [ "$\\displaystyle - \\frac{4 d^{2}}{fh^{2}} + 2$" ], "text/plain": [ " 2 \n", " 4⋅d \n", "- ──── + 2\n", " 2 \n", " fh " ] }, "execution_count": 4, "metadata": {}, "output_type": "execute_result" } ], "source": [ "trace_M = sp.simplify(M_cell[0,0] + M_cell[1,1])\n", "trace_M" ] }, { "cell_type": "markdown", "metadata": {}, "source": [ "#### Phase advance:\n", "$$\n", "\\cos \\phi_{cell} = \\frac{1}{2}Trace(M_{cell}) = \\frac{1}{2} \\Big(2 - \\frac{4 d^2}{f_h^2}\\Big) = 1 - \\frac{2 d^2}{f_h^2} = 1 - 2\\frac{L_{cell}^2}{16 f^2}\n", "$$\n", "\n", "using trigonometric double-angle formulae:\n", "$$\n", "\\cos 2\\phi = 1-2\\sin^2 \\phi\n", "$$\n", "and finally \n", "$$\n", "\\boxed{\\sin \\phi_{cell}/2 = \\frac{L_{cell}}{4f}}\n", "$$\n", "#### Stability \n", "$$\n", "\\left| 2 - \\frac{4 d^2}{f_h^2}\\right| < 2\n", "$$\n", "rewriting \n", "$$\n", "0 < \\frac{4 d^2}{f_h^2} < 4 \\qquad \\to \\qquad f_h > d\n", "$$\n", "Finally \n", "$$\n", "\\boxed{f > \\frac{L_{cell}}{4}}\n", "$$" ] }, { "cell_type": "markdown", "metadata": {}, "source": [ "### $\\beta$ - functions\n", "\n", "General solution of the Hill's equation $x'' + K(s)x = 0$:\n", "\n", "\\begin{equation}\n", "\\begin{cases}\n", "x(s) = \\sqrt{\\varepsilon \\beta(s)}\\cos(\\phi(s) + \\phi)\\\\\n", "x'(s) = -\\sqrt{\\frac{\\varepsilon}{\\beta(s)}}\\Big(\\alpha(s)\\cos(\\phi(s) + \\phi) + \\sin(\\phi(s) + \\phi)\\Big)\n", "\\end{cases}\n", "\\end{equation}\n", "$$$$\n", "Applying initial conditions at the point $s(0) = s_0$ with $\\phi(0) = 0$ and the particle coordinates $x_0, x'_0$, we will get:\n", "\n", "\\begin{equation}\n", "\\begin{cases}\n", "\\cos\\phi = \\frac{x_0}{\\sqrt{\\varepsilon \\beta_0}}\\\\\n", "\\sin\\phi = -\\frac{1}{\\sqrt{\\varepsilon}}\\Big( \\frac{\\alpha_0}{\\sqrt{\\beta_0}}x_0 + \\sqrt{\\beta_0}x'_0\\Big)\n", "\\end{cases}\n", "\\end{equation}\n", "inserting the result into the equation above:\n", "\\begin{equation}\n", "\\begin{bmatrix}\n", " x_1 \\\\\n", " x'_1 \n", "\\end{bmatrix} = \n", "\\begin{bmatrix}\n", " \\sqrt{\\frac{\\beta_1}{\\beta_0}}(\\cos\\phi_{0\\to1} + \\alpha_0\\sin\\phi_{0\\to1}) & \\sqrt{\\beta_1\\beta_0}\\sin\\phi_{0\\to1})\\\\\n", " \\frac{(\\alpha_0 - \\alpha_1)\\cos\\phi_{0\\to1} - (1 + \\alpha_0\\alpha_2)\\sin\\phi_{0\\to1}}{\\sqrt{\\beta_1\\beta_0}} & \\sqrt{\\frac{\\beta_0}{\\beta_1}}(\\cos\\phi_{0\\to1} - \\alpha_1\\sin\\phi_{0\\to1})\n", "\\end{bmatrix}\n", "\\cdot\n", "\\begin{bmatrix}\n", " x_0 \\\\\n", " x'_0 \n", "\\end{bmatrix}\n", "\\end{equation}\n", "$$$$\n", "Now consider half of the FODO cell (see picture above) ($\\phi_{0\\to1} = \\phi_{cell}/2$) and $\\beta_0 = \\beta_{min}$ and $\\beta_1 = \\beta_{max}$ (we consider horizontal plane) and remembering that in the middle of the quads $\\alpha = 0$. From another side, we have already calculated the transfer matrix for [a half of FODO cell](#half_cell). Collecting all together:\n", "\n", "\\begin{equation} \n", "\\begin{bmatrix}\n", " \\sqrt{\\frac{\\beta_{max}}{\\beta_{min}}}\\cos\\frac{\\phi_{cell}}{2} & \\sqrt{\\beta_{max}\\beta_{min}}\\sin\\frac{\\phi_{cell}}{2}\\\\\n", " -\\frac{\\sin\\frac{\\phi_{cell}}{2}}{\\sqrt{\\beta_{max}\\beta_{min}}} & \\sqrt{\\frac{\\beta_{min}}{\\beta_{max}}}\\cos\\frac{\\phi_{cell}}{2}\n", "\\end{bmatrix};\n", "\\qquad\n", "\\begin{bmatrix}\n", " 1 + d/f_h & d \\\\\n", " -d/f_h^2 & 1 - d/f_h \n", "\\end{bmatrix}\n", "\\end{equation}\n", "\n", "after some gymnastics with trigonometric identities (and taking into account $\\sin \\phi_{cell}/2 = \\frac{d}{f_h}= \\frac{L_{cell}}{4f}$)\n", "\n", "$$\n", "\\beta_{max} = \\frac{1 + \\sin \\frac{\\phi_{cell}}{2}}{\\sin\\phi_{cell}}L_{cell}\n", "$$\n", "\n", "$$\n", "\\beta_{min} = \\frac{1 - \\sin \\frac{\\phi_{cell}}{2}}{\\sin\\phi_{cell}}L_{cell}\n", "$$\n", "If we neglect the fact the beta function in the drift space takes the form of a quadratic function ($\\beta(s) = \\beta_0 - 2\\alpha_0 s + \\gamma_0 s^2$) and assume instead a linear behavior, the average $\\beta$-function will be\n", "$$\n", "\\overline \\beta \\approx \\frac{\\beta_{max} + \\beta_{min}}{2} = \\frac{L_{cell}}{\\sin\\phi_{cell}}\n", "$$\n", "\n", "What means that minimum average beta-function will be with phase advance $\\phi_{cell} = 90^0$. For SASE1 undulator $L_{cell} = 12.2$ and taking into account that our calculations were done with some level approximation:\n", "$$\n", "\\overline \\beta_{min} \\approx \\frac{12.2 m}{\\sin 90^0} = 12.2 m\n", "$$\n" ] }, { "cell_type": "markdown", "metadata": {}, "source": [ "### More accurate solution\n", "Taking into account quadratic shape of the $\\beta$ in the drift space and go through some simple steps, finally we will get more exact solution:\n", "1. Twiss parameters transformation \n", "\\begin{equation}\n", "\\begin{bmatrix}\n", " \\beta \\\\\n", " \\alpha \\\\\n", " \\gamma\n", "\\end{bmatrix}_1 = \n", "\\begin{bmatrix}\n", " C^2 & -2SC & S^2\\\\\n", " -C C' & SC' + S'C & -SS'\\\\\n", " C'^2 & -2 S'C' & S'^2\n", "\\end{bmatrix}\n", "\\cdot\n", "\\begin{bmatrix}\n", " \\beta \\\\\n", " \\alpha \\\\\n", " \\gamma\n", "\\end{bmatrix}_0\n", "\\end{equation}\n", "Where $C,S, C', S'$ are elements of the transfer matrix\n", "\\begin{equation}\n", "M = \n", "\\begin{bmatrix}\n", " C & S \\\\\n", " C' & S' \n", "\\end{bmatrix}\n", "\\end{equation}\n", "\n", "2. twiss parameters after the thin quadrupole:\n", "$$\n", "\\beta_1 = \\beta_0 \\\\\n", "\\alpha_1 = -\\frac{\\beta_0}{f} + \\alpha_0 \\\\\n", "\\gamma_1 = \\frac{1}{f^2} - 2 \\frac{\\alpha_0}{f} + \\gamma_0\n", "$$\n", "In the middle of the quadrupole $\\alpha_0 = 0$ then\n", "$$\n", "\\alpha_1 = -\\frac{\\beta_0}{f_h}\n", "$$\n", "3. In the drift space $\\beta$ behave:\n", "$$\n", "\\beta(s) = \\beta_0 - 2\\alpha_0 s + \\gamma_0 s^2\n", "$$\n", "4. Because FODO cell is symetric and we are considering the thin lens approximation the average $\\overline \\beta$ is:\n", "$$\n", "\\overline \\beta = \\frac{2}{L}\\int_0^{L/2}\\left(\\beta_0 - 2\\alpha_0 s + \\gamma_0s^2 \\right)ds = \\beta_0 - \\frac{\\alpha_0 L}{2} + \\gamma_0\\frac{L^2}{12}\n", "$$\n", "5. Inserting expression for $\\beta_{min}$ and express focal length against the phase advance we will get:\n", "$$\n", "\\boxed{\\overline \\beta = \\frac{L}{6 \\sin\\phi_{cell}}\\big(5 + \\cos\\phi_{cell} \\big)}\n", "$$\n", "\n", "
\n", "Note: The result looks simple, probably this expression can be found in some handbooks. \n", "
\n", "\n", "\n", "$$$$\n", "The minimum possible average $\\overline\\beta_{min}$ will be with $\\frac{d \\overline \\beta}{d\\phi_{cell}} = 0$ what gives us $\\phi_{cell} = \\pi - \\arctan 2\\sqrt 6 \\approx 101^0$. \n", "\n", "And for our case $L_{cell} = 12.2 m$ it will be \n", "$$\n", "\\overline \\beta_{min} \\approx 9.96\n", "$$" ] }, { "cell_type": "markdown", "metadata": {}, "source": [ "Might be useful to find condition for minimum possible $\\beta_{max}$ \n", "$$\n", "\\frac{d}{d\\phi}\\beta_{max} = 0\n", "$$" ] }, { "cell_type": "code", "execution_count": 5, "metadata": {}, "outputs": [ { "data": { "image/png": "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", "text/latex": [ "$\\displaystyle - \\frac{\\left(\\sin{\\left(\\frac{\\phi}{2} \\right)} + 1\\right) \\cos{\\left(\\phi \\right)}}{\\sin^{2}{\\left(\\phi \\right)}} + \\frac{\\cos{\\left(\\frac{\\phi}{2} \\right)}}{2 \\sin{\\left(\\phi \\right)}}$" ], "text/plain": [ " ⎛ ⎛φ⎞ ⎞ ⎛φ⎞ \n", " ⎜sin⎜─⎟ + 1⎟⋅cos(φ) cos⎜─⎟ \n", " ⎝ ⎝2⎠ ⎠ ⎝2⎠ \n", "- ─────────────────── + ────────\n", " 2 2⋅sin(φ)\n", " sin (φ) " ] }, "execution_count": 5, "metadata": {}, "output_type": "execute_result" } ], "source": [ "phi = sp.Symbol(\"phi\")\n", "sp.diff((1 + sp.sin(phi/2))/sp.sin(phi), phi)" ] }, { "cell_type": "code", "execution_count": 6, "metadata": {}, "outputs": [ { "name": "stdout", "output_type": "stream", "text": [ "Cell phase advance for minimum possible bmax: phi = [76.34541525]\n" ] } ], "source": [ "import numpy as np\n", "from scipy.optimize import root\n", "from matplotlib import pyplot as plt\n", "fun = lambda phi: -(np.sin(phi/2) + 1)*np.cos(phi)/np.sin(phi)**2 + np.cos(phi/2)/(2*np.sin(phi))\n", "res = root(fun, 0.1)\n", "print(\"Cell phase advance for minimum possible bmax: phi = \", res.x*180/np.pi)" ] }, { "cell_type": "markdown", "metadata": {}, "source": [ "### SASE1 undulator. Numerical simulation\n" ] }, { "cell_type": "code", "execution_count": 7, "metadata": {}, "outputs": [ { "name": "stdout", "output_type": "stream", "text": [ "initializing ocelot...\n" ] }, { "data": { "image/png": "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", "text/plain": [ "
" ] }, "metadata": {}, "output_type": "display_data" } ], "source": [ "%matplotlib inline\n", "from ocelot import *\n", "from ocelot.gui.accelerator import *\n", "from scipy.integrate import simps\n", "from ocelot.cpbd import optics\n", "\n", "d1 = Drift(l=0.43065, eid='d1')\n", "d2 = Drift(l=0.55565, eid='d2')\n", "l = 0.1137\n", "k = 2*0.1137/l\n", "\n", "qd = Quadrupole(l=l/2, k1=-k, tilt=0.0)\n", "qf = Quadrupole(l=l, k1=k, tilt=0.0)\n", "K = 3\n", "u40 = Undulator(lperiod=0.04, nperiods=125, Kx=K, Ky=0.0)\n", "m1 = Marker()\n", "m2 = Marker()\n", "fodo_cell = [m1, qd, d1, u40, d2, qf, d1, u40, d2, qd, m2]\n", "fodo_lat = MagneticLattice(fodo_cell)\n", "\n", "tws = twiss(fodo_lat, nPoints=1000)\n", "\n", "plot_opt_func(fodo_lat, tws, top_plot=[\"mux\", \"muy\"], legend=False)\n", "plt.show()" ] }, { "cell_type": "code", "execution_count": 8, "metadata": {}, "outputs": [ { "name": "stdout", "output_type": "stream", "text": [ "min(beta_av) = 9.991835898036324 m\n" ] }, { "data": { "image/png": "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", "text/plain": [ "
" ] }, "metadata": {}, "output_type": "display_data" } ], "source": [ "def beta(k):\n", " qd.k1 = -k\n", " qf.k1 = k\n", " fodo_lat.update_transfer_maps()\n", " tws = twiss(fodo_lat, nPoints=1000)\n", " bx = np.array([tw.beta_x for tw in tws])\n", " s = np.array([tw.s for tw in tws])\n", " bx_av = simps(bx, s)/fodo_lat.totalLen\n", " phi = tws[-1].mux\n", " L = fodo_lat.totalLen\n", " bx_av_th = L/6 * (5 + np.cos(phi))/np.sin(phi)\n", " return bx_av, bx.max(), bx.min(), tws[-1].mux*180/np.pi, bx_av_th\n", "\n", "k = np.arange(0.3, 2.9, 0.05)\n", "Bx_min = []\n", "Bx_max = []\n", "Bx_av = []\n", "Bx_av_theory = []\n", "Phi = []\n", "for k1 in k:\n", " bx_av, bx_max, bx_min, phi, bx_av_th = beta(k1)\n", " Bx_min.append(bx_min)\n", " Bx_max.append(bx_max)\n", " Bx_av.append(bx_av)\n", " Phi.append(phi)\n", " Bx_av_theory.append(bx_av_th)\n", " \n", "fig, ax1 = plt.subplots()\n", "plt.title(r\"$\\beta$ - average and max against quad strength\")\n", "ax1.plot(k, Bx_av, label=r\"$\\beta_{av}$\")\n", "ax1.plot(k, Bx_av_theory,\"g--\", label=r\"$\\beta_{av}^{theory}$\")\n", "ax1.plot(k, Bx_max, label=r\"$\\beta_{max}$\")\n", "ax1.set_ylabel(r\"$\\beta$, m\")\n", "ax1.set_xlabel(\"k, 1/m2\")\n", "plt.legend()\n", "print(\"min(beta_av) = \", np.min(Bx_av), \"m\" )\n", "ax2 = ax1.twinx()\n", "ax2.plot(k, Phi, \"r\", label=r\"$\\phi_{cell}$\")\n", "ax2.set_ylabel(r\"$\\phi_{cell}$\", color='r')\n", "ax2.tick_params('y', colors='r')\n", "plt.legend()\n", "plt.grid(False)\n", "plt.show()" ] }, { "cell_type": "markdown", "metadata": {}, "source": [ "### Limitations\n", "\n", "The maximum possible strength of the undulator quadrupoles is $k_{max} = 1.94729$ what means that we can have minimum average beta-function close to 10m (see the Figure above). \n", "\n", "Now we need to consider the limitations of the matching sections." ] }, { "cell_type": "markdown", "metadata": {}, "source": [ "## Inverse the task\n", "\n", "Find the quad strength and FODO cell params if beta average is defined\n", "\n", "### FODO estimator" ] }, { "cell_type": "code", "execution_count": 9, "metadata": {}, "outputs": [ { "name": "stdout", "output_type": "stream", "text": [ "k1 = 1.771751963648942 1/m^2\n", "bmin / bmax = 4.851965262428619 / 20.314939511625177 m\n", "calculated beta_av = 11.000000000000059 m\n", "phi = 75.8191420392388 deg\n" ] } ], "source": [ "from scipy.optimize import root\n", "\n", "def fodo_estimator(beta_av, Lcell, lq=0.1137):\n", " fun = lambda phi: Lcell/6 * (5 + np.cos(phi))/np.sin(phi) - beta_av\n", " res = root(fun, 0.1)\n", " phi = res.x[0]\n", " b_av = Lcell/6 * (5 + np.cos(phi))/np.sin(phi)\n", " f = Lcell/(4*np.sin(phi/2))\n", " kq = 1/f/lq\n", " bmax = (1 + np.sin(phi/2))/np.sin(phi)*Lcell\n", " bmin = (1 - np.sin(phi/2))/np.sin(phi)*Lcell\n", " return kq, phi, bmin, bmax, b_av\n", " \n", "kq, phi, bmin, bmax, b_av = fodo_estimator(beta_av=11, Lcell=12.2)\n", "print(\"k1 = \", kq, \"1/m^2\")\n", "print(\"bmin / bmax = \", bmin, \"/\", bmax, \"m\")\n", "print(\"calculated beta_av = \", b_av, \"m\")\n", "print(\"phi = \", phi*180/np.pi, \"deg\")" ] }, { "cell_type": "markdown", "metadata": {}, "source": [ "### FODO correction due to undulator focusing " ] }, { "cell_type": "code", "execution_count": 10, "metadata": {}, "outputs": [], "source": [ "def fodo_correction_SASE1(beta_av=60, K=3):\n", " kq, phi, bmin, bmax, b_av = fodo_estimator(beta_av=beta_av, Lcell=12.2)\n", " \n", " print(\"Estimation: beta_x = \", np.round(bmin,3), \"; beta_y = \", np.round(bmax, 3),\n", " \"; phi = \", np.round(phi*180/np.pi), \"grad\"\n", " \"; kf/kd = \", np.round(kq, 4),\"/\", np.round(-kq, 4))\n", " \n", " # SASE1 FODO cell\n", " d1 = Drift(l=0.43065, eid='d1')\n", " d2 = Drift(l=0.55565, eid='d2')\n", " qd = Quadrupole(l=0.1137/2, k1=-kq, tilt=0.0)\n", " qf = Quadrupole(l=0.1137, k1=kq, tilt=0.0)\n", " u40 = Undulator(lperiod=0.04, nperiods=125, Kx=K, Ky=0.0)\n", " m1 = Marker()\n", " m2 = Marker()\n", " fodo_cell = [m1, qd, d1, u40, d2, qf, d1, u40, d2, qd, m2]\n", " \n", " # constraints\n", " constr = {fodo_cell[-1]: {'mux':phi, \"muy\":phi}, \"periodic\": True}\n", "\n", " # variables\n", " vars = [qf, qd]\n", " \n", " tws = Twiss()\n", " tws.beta_x = bmin\n", " tws.beta_y = bmax\n", " tws.E = 14\n", " \n", " res = match(MagneticLattice(fodo_cell), constr=constr,tw=tws, \n", " vars=vars, max_iter=2000, verbose=False)\n", " kf, kd = res\n", " \n", " qf.k1 = kf\n", " qd.k1 = kd\n", " \n", " fodo_lat = MagneticLattice(fodo_cell)\n", "\n", " tws = twiss(fodo_lat, tws0=tws, nPoints=1000)\n", " print(\"Correction: beta_x = \", np.round(tws[0].beta_x, 3), \n", " \"beta_y = \", np.round(tws[0].beta_y, 3), \n", " \" kf/kd = \", np.round(kf,4), \"/\", np.round(kd,4))\n", " #plot_opt_func(fodo_lat, tws, top_plot=[\"mux\", \"muy\"], legend=False)\n", " #plt.show()\n", " return kf, kd, bmin, bmax" ] }, { "cell_type": "markdown", "metadata": {}, "source": [ "### Current lattice $\\overline\\beta \\approx 32$ m" ] }, { "cell_type": "code", "execution_count": 11, "metadata": {}, "outputs": [], "source": [ "import sase1_lattice as sase1" ] }, { "cell_type": "code", "execution_count": 12, "metadata": {}, "outputs": [ { "name": "stdout", "output_type": "stream", "text": [ "emit_x = 0.0\n", "emit_y = 0.0\n", "beta_x = 10.506745988156398\n", "beta_y = 42.02704133328497\n", "alpha_x = 0.676085898798978\n", "alpha_y = -2.1449229692237783\n", "gamma_x = 0.0\n", "gamma_y = 0.0\n", "Dx = 0.0\n", "Dy = 0.0\n", "Dxp = 0.0\n", "Dyp = 0.0\n", "mux = 0.0\n", "muy = 0.0\n", "nu_x = 0.0\n", "nu_y = 0.0\n", "E = 14\n", "s = 0.0\n", "\n" ] }, { "data": { "image/png": "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", "text/plain": [ "
" ] }, "metadata": {}, "output_type": "display_data" } ], "source": [ "lat = MagneticLattice(sase1.cell)#, stop=sase1.fodo_match)\n", "print(sase1.tws)\n", "#sase1.tws.beta_x = 15\n", "#sase1.tws.beta_y = 50\n", "tws = twiss(lat, tws0=sase1.tws)\n", "plot_opt_func(lat, tws, legend=False)\n", "plt.show()" ] }, { "cell_type": "markdown", "metadata": {}, "source": [ "### $\\overline \\beta = 60$ m\n", "\n", "### *Sometimes require run this cell two times to get a correct result*" ] }, { "cell_type": "code", "execution_count": 21, "metadata": {}, "outputs": [ { "name": "stdout", "output_type": "stream", "text": [ "Estimation: beta_x = 54.076 ; beta_y = 66.34 ; phi = 12.0 grad; kf/kd = 0.2937 / -0.2937\n", "initial value: x = [0.29368260023214693, -0.29368260023214693]\n", "Optimization terminated successfully.\n", " Current function value: 0.000032\n", " Iterations: 40\n", " Function evaluations: 79\n", "Correction: beta_x = 54.076 beta_y = 66.34 kf/kd = 0.2648 / -0.2583\n", "initial value: x = [-0.2244, 0.2309, -0.1911, 0.1653, -0.0881, 0.0953]\n", "Optimization terminated successfully.\n", " Current function value: 0.000031\n", " Iterations: 158\n", " Function evaluations: 279\n" ] }, { "data": { "image/png": "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", "text/plain": [ "
" ] }, "metadata": {}, "output_type": "display_data" } ], "source": [ "kf, kd, bmin, bmax = fodo_correction_SASE1(beta_av=60, K=3)\n", "# constraints\n", "constr = {sase1.fodo_match: {'beta_x':bmax, \"beta_y\":bmin, \n", " \"alpha_x\": 0, \"alpha_y\": 0}}\n", "\n", "quad_limits = {sase1.qf_2177_t2: [-0.654, 0],\n", " sase1.qf_2192_t2: [0 ,0.654],\n", " sase1.qf_2207_t2: [-0.654, 0],\n", " sase1.qf_2218_t2: [0 ,0.654],\n", " sase1.qa_2229_t2: [-1.94, 0],\n", " sase1.qa_2235_t2: [0, 1.94]}\n", "# variables\n", "vars = [sase1.qf_2177_t2, sase1.qf_2192_t2,\n", " sase1.qf_2207_t2, \n", " sase1.qf_2218_t2, sase1.qa_2229_t2, sase1.qa_2235_t2]\n", "\n", "# because of variables redundancy, we can help a bit to find a more elegant solution \n", "# *** comment lines or change inital conditions if you want to play with matching\n", "sase1.qf_2177_t2.k1 = -0.2244\n", "sase1.qf_2192_t2.k1 = 0.2309\n", "sase1.qf_2207_t2.k1 = -0.1911\n", "sase1.qf_2218_t2.k1 = 0.1653\n", "sase1.qa_2229_t2.k1 = -0.0881\n", "sase1.qa_2235_t2.k1 = 0.0953\n", "\n", "# *** comment lines or change inital conditions if you want to play with matching\n", "lat.update_transfer_maps()\n", "\n", "res = match(lat, constr=constr, vars=vars, tw=sase1.tws, max_iter=2000, verbose=False)\n", "\n", "sase1.qa_2241_sa1.k1 = kd\n", "sase1.qa_2247_sa1.k1 = kf\n", "sase1.qa_2247_sa1_h.k1 = kf\n", "lat.update_transfer_maps()\n", "tws = twiss(lat, tws0=sase1.tws)\n", "plot_opt_func(lat, tws, legend=False)\n", "plt.show()" ] }, { "cell_type": "markdown", "metadata": {}, "source": [ "#### Checking quads limits" ] }, { "cell_type": "code", "execution_count": 22, "metadata": {}, "outputs": [ { "name": "stdout", "output_type": "stream", "text": [ "QF.2177.T2.k1 = -0.2244; strength is OK : True\n", "QF.2192.T2.k1 = 0.2309; strength is OK : True\n", "QF.2207.T2.k1 = -0.1911; strength is OK : True\n", "QF.2218.T2.k1 = 0.1653; strength is OK : True\n", "QA.2229.T2.k1 = -0.0881; strength is OK : True\n", "QA.2235.T2.k1 = 0.0953; strength is OK : True\n" ] } ], "source": [ "# check quads limits\n", "for q, k in zip(vars, res):\n", " print(q.id + \".k1 = \"+ str(np.round(k,4)) + \"; strength is OK :\",\n", " quad_limits[q][0]" ] }, "metadata": {}, "output_type": "display_data" } ], "source": [ "kf, kd, bmin, bmax = fodo_correction_SASE1(beta_av=11, K=3)\n", "# constraints\n", "constr = {sase1.fodo_match: {'beta_x':bmax, \"beta_y\":bmin, \n", " \"alpha_x\": 0, \"alpha_y\": 0}}\n", "\n", "\n", "# variables\n", "vars = [sase1.qf_2177_t2, sase1.qf_2192_t2, sase1.qf_2207_t2, \n", " sase1.qf_2218_t2, sase1.qa_2229_t2, sase1.qa_2235_t2]\n", "\n", "# because of variables redundancy, we can help a bit to find a more elegant solution\n", "# *** comment lines or change inital conditions if you want to play with matching\n", "sase1.qf_2177_t2.k1 = -0.2227\n", "sase1.qf_2192_t2.k1 = 0.211\n", "sase1.qf_2207_t2.k1 = -0.2176\n", "sase1.qf_2218_t2.k1 = 0.2392\n", "sase1.qa_2229_t2.k1 = -0.9728\n", "sase1.qa_2235_t2.k1 = 1.3215\n", "# *** comment lines or change inital conditions if you want to play with matching\n", "lat.update_transfer_maps()\n", "\n", "res = match(lat, constr=constr, vars=vars, tw=sase1.tws, max_iter=2000, verbose=False)\n", "\n", "sase1.qa_2241_sa1.k1 = kd\n", "sase1.qa_2247_sa1.k1 = kf\n", "sase1.qa_2247_sa1_h.k1 = kf\n", "lat.update_transfer_maps()\n", "tws = twiss(lat, tws0=sase1.tws)\n", "plot_opt_func(lat, tws, legend=False)\n", "plt.show()" ] }, { "cell_type": "markdown", "metadata": {}, "source": [ "#### Checking quads limits" ] }, { "cell_type": "code", "execution_count": 25, "metadata": {}, "outputs": [ { "name": "stdout", "output_type": "stream", "text": [ "QF.2177.T2.k1 = -0.2226; strength is OK : True\n", "QF.2192.T2.k1 = 0.211; strength is OK : True\n", "QF.2207.T2.k1 = -0.2176; strength is OK : True\n", "QF.2218.T2.k1 = 0.2393; strength is OK : True\n", "QA.2229.T2.k1 = -0.9747; strength is OK : True\n", "QA.2235.T2.k1 = 1.3232; strength is OK : True\n" ] } ], "source": [ "# check quads limits\n", "for q, k in zip(vars, res):\n", " print(q.id + \".k1 = \"+ str(np.round(k,4)) + \"; strength is OK :\",\n", " quad_limits[q][0]